• Shuffle
    Toggle On
    Toggle Off
  • Alphabetize
    Toggle On
    Toggle Off
  • Front First
    Toggle On
    Toggle Off
  • Both Sides
    Toggle On
    Toggle Off
  • Read
    Toggle On
    Toggle Off
Reading...
Front

Card Range To Study

through

image

Play button

image

Play button

image

Progress

1/28

Click to flip

Use LEFT and RIGHT arrow keys to navigate between flashcards;

Use UP and DOWN arrow keys to flip the card;

H to show hint;

A reads text to speech;

28 Cards in this Set

  • Front
  • Back
942. Although ß-adrenergic receptor blockade is the best treatment for reentrant tachydysrhythmia associated with Romano-Ward syndrome, these dysrhythmias can also be effectively treated with
...A. Lidocaine
...B. Procainamide
...C. Quinidine
...D. Left stellate ganglion blockade
...E. Right stellate ganglion blockade
942. (D)
943. A 64-year-old patient with an axial flow left ventricular assist device (HeartMate II, Jarvik 2000) is scheduled for laparoscopic cholecystectomy under general anesthesia. Monitoring which of the following parameters is likely to be difficult in this patient?
...A. Blood pressure with blood pressure cuff
...B. Blood pressure with arterial line
...C. Pulmonary artery pressure with pulmonary artery (PA) catheter
...D. Temperature with esophageal temperature probe
...E. End tidal isoflurane concentration with mass spectrometer
943. (A)
944. In a normal person, what percentage of the cardiac output is dependent on the “atrial kick”?
...A. 25%
...B. 35%
...C. 45%
...D. 55%
...E. 65%
944. (A)
945. This arterial waveform is consistent with
...A. Aortic regurgitation
...B. Aortic stenosis
...C. Cardiac tamponade
...D. Hypovolemia
...E. Severe diastolic dysfunction
945. (A)
946. A 1-year-old child with tetralogy of Fallot is to undergo elective repair of a left inguinal hernia under general anesthesia. Which of the following anesthetics would provide the most stable hemodynamics in this patient?
...A. Sevoflurane & N2O
...B. Isoflurane & N2O
...C. Desflurane & oxygen
...D. Fentanyl & N2O
...E. Ketamine
946. (E)
947. The left ventricular pressure-volume loop shown in the figure depicts
...A. Mitral stenosis
...B. Mitral regurgitation
...C. Aortic stenosis
...D. Acute aortic insufficiency
...E. Chronic aortic insufficiency
947. (A)
948. A 54-year-old patient is undergoing a three-vessel coronary artery bypass graft under general anesthesia. After induction, the pulmonary capillary wedge pressure is 15 mm Hg & pulmonary artery pressures are 26/13 mm Hg. Suddenly, new 30-mm Hg V waves appear on the monitor screen. Systemic blood pressure is 120/70 mm Hg, heart rate is 75 beats/min, & pulmonary artery pressure is 50/35 mm Hg. Which of the following drugs should be administered to the patient?
...A. Nitroglycerin
...B. Nitroprusside
...C. Esmolol
...D. Phenylephrine
...E. Dobutamine
948. (A)
949. A 62-year-old patient scheduled for elective repair of an abdominal aortic aneurysm develops a wide complex regular tachycardia (heart rate 150) during induction of anesthesia. Blood pressure is 110/78. Which of the following drugs would be most useful in the management of this dysrhythmia?
...A. Lidocaine, 100 mg IV
...B. Amiodarone, 150 mg IV over 10 minutes
...C. Adenosine, 6 mg rapidly over 3 seconds
...D. Verapamil, 5 to 10 mg IV
...E. Esmolol, 35 mg IV
949. (B)
950. Under maximum stress, how much cortisol is produced per day?
...A. 50 mg
...B. 150 mg
...C. 250 mg
...D. 350 mg
...E. Up to 1000 mg
950. (B)
951. A VVI pacemaker programmed to pace at a rate of 70 beats/min is noted on the preoperative ECG to pace at 61 beats/min. The most likely reason for this decrease in the pacing heart rate is
...A. Decreased atrial rate
...B. Third-degree heart block
...C. Trifascicular heart block
...D. Battery failure
...E. Normal variation
951. (D)
952. Calculate the cardiac output from the following data: patient weight 70 kg, hemoglobin concentration 10 mg/dL, arterial blood gases on 100% O2: Pao2 450 mm Hg, Paco2 32 mm Hg, pH 7.46, Sao2 99%. Mixed venous blood gases are: Pvo2 30 mm Hg, Paco2 45 mm Hg, pH 7.32, Svo2 60%.
...A. 2.5 L/min
...B. 3.0 L/min
...C. 3.5 L/min
...D. 4.0 L/min
...E. 4.5 L/min
952. (E)
953. Normal resting myocardial O2 consumption is
...A. 2.0 mL/100 g/min
...B. 3.5 mL/100 g/min
...C. 10 mL/100 g/min
...D. 15 mL/100 g/min
...E. 25 mL/100 g/min
953. (C)
954. A 22-year-old man with hypertrophic cardiomyopathy (HOCM) is undergoing an elective cholecystectomy under general anesthesia. Immediately after induction with thiopental, 5 mg/kg IV, the arterial blood pressure decreases from 140/82 to 70/40 mm Hg. What would be the most appropriate drug for treatment of hypotension in this patient?
...A. Ephedrine
...B. Mephentermine
...C. Isoproterenol
...D. Phenylephrine
...E. Epinephrine
954. (D)
955. A 65-year-old patient with moderate aortic stenosis develops a sudden increase in heart rate during an appendectomy under general anesthesia. The ventricular rate is 190 beats/min & is irregularly irregular, arterial blood pressure is 70/45 mm Hg, & there is 2 mm ST-segment depression in lead V5 of the ECG. Which of the following would be the most appropriate treatment for myocardial ischemia in this patient?
...A. Electrical cardioversion
...B. Esmolol
...C. Nitroglycerin
...D. Verapamil
...E. Phenylephrine
955. (A)
956. After emergency repair of a ruptured abdominal aortic aneurysm, a 68-year-old patient is mechanically ventilated in the intensive care unit with 20 cm H2O of positive end-expiratory pressure (PEEP) for 3 days. Sodium nitroprusside has been infused at a rate of 1.5 μg/kg/min for 48 hours to control hypertension. Suddenly, the systemic blood pressure falls from 130/70 to 50 mm Hg systolic & the Sao2 drops to 75%. The most likely cause of this scenario is
...A. Cyanide toxicity
...B. Acute myocardial infarction
...C. Tension pneumothorax
...D. Hyperventilation
...E. Methemoglobinemia
956. (C)
957. Normal resting coronary artery blood flow is
...A. 10 mL/100 g/min
...B. 40 mL/100 g/min
...C. 75 mL/100 g/min
...D. 120 mL/100 g/min
...E. 160 mL/100 g/min
957. (C)
958. Each of the following is associated with an increased incidence of pulmonary artery rupture in patients with pulmonary artery catheters EXCEPT
...A. Hypothermia
...B. Presence of pulmonary artery atheromas
...C. Old age
...D. Anticoagulation
...E. Pulmonary artery catheter migration
958. (B)
959. Allergic reactions to protamine can occur with each of the following EXCEPT?
...A. Diabetes treated with NPH insulin
...B. Diabetes treated with regular insulin
...C. Diabetics treated with PZI insulin
...D. Previous vasectomy
...E. Allergy to seafood
959. (B)
960. A 66-year-old patient is undergoing a three-vessel coronary artery bypass operation. Anticoagulation is achieved with 20,000 units of heparin. How much protamine should be administered to this patient to completely reverse the heparin after cardiopulmonary bypass?
...A. 150 mg
...B. 250 mg
...C. 350 mg
...D. 450 mg
...E. 550 mg
960. (B)
961. The graph below represents
...A. Ventricular end-diastolic pressure as a function of ventricular end-diastolic volume
...B. Stroke volume as a function of end-diastolic pressure
...C. Cardiac index as a function of end-diastolic pressure
...D. Cardiac output as a function of ventricular end-diastolic volume
...E. Diastolic time (as percentage of cardiac cycle) as a function of heart rate
961. (E)
962. A 72-year-old woman is undergoing cardiopulmonary bypass for aortic & mitral valve replacement. The surgery is uneventful; however, in the intensive care unit, blood is noted to ooze from the pulmonary artery catheter & venous access sites. Mediastinal chest tube output is 500 mL/hour. A thromboelastogram is obtained & shown in the figure. What is the most likely cause of profuse bleeding in this patient?
...A. Fibrinolysis
...B. Excess heparin
...C. Thrombocytopenia
...D. Factor VIII deficiency
...E. Poor surgical hemostasis
962. (A)
963. A 170-micrometer filter must be used for administration of each of the following EXCEPT
...A. Fresh frozen plasma
...B. Cryoprecipitate
...C. Platelets
...D. Packed red cells
...E. Albumin
963. (E)
964. The dose of adenosine necessary to convert paroxysmal supraventricular tachycardia to normal sinus rhythm should be initially reduced
...A. In patients receiving theophylline for chronic asthma
...B. In patients with a history of arterial thrombotic disease taking dipyridamole
...C. In patients with a history of chronic renal failure
...D. In patients with hepatic dysfunction
...E. In chronic alcoholics
964. (B)
965. A 56-year-old male patient is anesthetized for elective coronary revascularization. A urinary catheter is placed after induction & coupled to a temperature transducer. A pulmonary artery catheter is inserted, & the temperature probe on the distal portion of the catheter is also connected to a transducer. The reason for measuring the temperature of both the bladder & the blood in the pulmonary vasculature is
...A. Both are necessary for determining cardiac output by the thermodilution technique
...B. Bladder temperature is more accurate prebypass; pulmonary artery catheter temperature is more accurate postbypass
...C. Pulmonary artery catheter temperature is more accurate prebypass; bladder temperature is more accurate postbypass
...D. It is helpful in determining the likelihood of recooling after discontinuation of cardiopulmonary bypass
...E. It is the average of these two temperatures, which is important in determining patient body warmth
965. (D)
966. Which of the following would be the best intraoperative transesophageal echocardiograph (TEE) view to monitor for myocardial ischemia?
...A. Mid-esophageal 4 chamber view
...B. Transgastric mid-papillary left ventricular short axis view
...C. Mid-esophageal long axis view
...D. Mid-esophageal 2 chamber view
...E. Transgastric 2 chamber view
966. (B)
967. Select the true statement regarding CPR & defibrillation by a health care provider in patients experiencing sudden cardiac arrest
...A. Defibrillation times one should always precede CPR
...B. CPR should always be carried out for 2 minutes prior to defibrillation
...C. Two minutes of chest compressions alone (no ventilation) should be carried out prior to first shock
...D. If arrest less than 1 minute (witnessed) one biphasic shock then 5 cycles of CPR
...E. If arrest less than 1 minute (witnessed) three biphasic shocks then 5 cycles of CPR
967. (D)
968. Which of the following medications blocks angiotensin at the receptor?
...A. Losartan (Cozaar)
...B. Terazosin (Hytrin)
...C. Lisinopril (Prinivil, Zestril)
...D. Spironolactone (Aldactone)
...E. Amlodipine (Norvasc)
968. (A)
969. Untoward effects associated with administration of sodium bicarbonate during massive blood transfusion include each of the following EXCEPT
...A. Hyperosmolality
...B. Paradoxical cerebrospinal fluid acidosis
...C. Hypercarbia
...D. Hypernatremia
...E. Hyperkalemia
969. (E)